subject
Mathematics, 22.04.2020 01:36 anthonysf

Using strong induction (so, neither weak nor "weak++") on the number of matches in the pile, prove that if this number is of the form 4k + 1, for k ∈ N, then P2 always has a strategy to win (which they will follow, because they are smart). Otherwise, P1 always have a strategy to win (which they will also follow because they’re smart too)! To be clear, you will need to prove both facts with a single inductive proof! That is to say, you will need to prove that P2 can always win under the aforementioned condition, but whenever that condition is not met, you have to prove that it is now P1 that can always win!

ansver
Answers: 2

Another question on Mathematics

question
Mathematics, 21.06.2019 12:30
Nparallelogram lmno, what are the values of x and y? x = 11, y = 14 x = 11, y = 25 x = 55, y = 14 x = 55, y = 25n parallelogram lmno, what are the values of x and y? x = 11, y = 14 x = 11, y = 25 x = 55, y = 14 x = 55, y = 25
Answers: 2
question
Mathematics, 21.06.2019 15:00
Two lines parallel to a third line are parallel to each other. always sometimes or never
Answers: 1
question
Mathematics, 21.06.2019 19:00
Billy plotted −3 4 and −1 4 on a number line to determine that −3 4 is smaller than −1 4 .is he correct? explain why or why not
Answers: 3
question
Mathematics, 21.06.2019 20:00
Someone me! if you’re smart you can solve this!
Answers: 2
You know the right answer?
Using strong induction (so, neither weak nor "weak++") on the number of matches in the pile, prove t...
Questions
question
Mathematics, 28.11.2020 05:20
question
Mathematics, 28.11.2020 05:20
Questions on the website: 13722363